find the sum of the series 9+3+1+........+1/27​

Answers

Answer 1

Answer:

364/27

Step-by-step explanation:

9, 3, 1, ... is a geometric sequence with common ratio r = 1/3.

 

The nth term is a1rn-1.

 

If a1rn-1 = 1/27, then 9(1/3)n-1 = 1/27

 

                                32(3-1)n-1 = 3-3

 

                                3231-n = 3-3

 

                                    33-n = 3-3

 

                                      3-n = -3

 

                                         n = 6

 

The sum, Sn, of the first n terms of a geometric sequence is given by Sn = a1(1 - rn) / (1 - r).

 

So, 9 + 3 + 1 + ... + 1/27 = S6 = 9(1 - (1/3)6) / (1 - 1/3)

 

                                              = 9(1 - 1/729) / (2/3)

 

                                              = (27/2)(728/729) = 364/27


Related Questions

Write a slope intercept equation for line with the given characteristics m=-3/5 passes through (4,-4)

Answers

Step-by-step explanation:

Hey there!

Given;

Slope (m) = -3/5

And passes through the point(4,-4).

Now; The equation of a line in slope intercept form is y= mx+b.

So, try to bring it in y= mx+b form.

Firstly let's use one-point formula.

[tex](y - y1) = m(x - x1)[/tex]

Now, keep all values.

[tex](y + 4) = \frac{ - 3}{5} (x - 4)[/tex]

Now, simplify to get answer.

[tex]5(y + 4) = - 3(x - 4)[/tex]

[tex]5y + 20 = - 3x + 12[/tex]

[tex]5y = - 3x + 12 - 20[/tex]

[tex]5y = - 3x - 8[/tex]

[tex]y = \frac{ - 3}{5} x - \frac{8}{5} [/tex]

Therefore the equation is y = -3/5x - 8/5.

Hope it helps...

Write an equation that relates the sale price, s, to the original price p.

Answers

Answer:

s= px

Step-by-step explanation:

3 + 15 + 3
- 4 2
I need help figuring this out

Answers

Answer:

3+15+3-42

=18+3-42

=21-42

=-21

A jet ski company charges a flat fee of $22.00 plus $3.00 per hour to rent a jet ski. Another company charges a fee of $17.50 plus $3.75 per hour to rent the same jet ski. Write a system of equations that can be used to determine when the cost will be the same, then solve your system.

Answers

Answer: The cost will be same for 6 hours.

Step-by-step explanation:

Given: A jet ski company charges a flat fee of $22.00 plus $3.00 per hour to rent a jet ski.

Another company charges a fee of $17.50 plus $3.75 per hour to rent the same jet ski.

Let x = Number of hours

y = Total cost

Then, Total cost (in $) for first company (y) = 22+3x

Total cost (in $) for second company (y) = 17.50+3.75x

So required system of equations :

y =  22+3x

y= 17.50+3.75x

When the cost will be the same,

22+3x = 17.50+3.75x

⇒ 3x - 3.75x = 17.50- 22

⇒ - 0.75x = -4.5

⇒ x =6  [Divide both sides by -0.75]

Hence,  the cost will be same for 6 hours.

x2 + 45x = -200 Using the quadratic formual and the discirimnat

Answers

Answer:

Positive discriminant = 2 real solution

x= -5,-40

Step-by-step explanation:

The discriminant is used to see how many solutions an equation has. If it is negative, the equation has no real solutions, if =0 the equation has 1, and if it is positive, the equation has two real solutions.

The discriminant is the part of the quadratic formula inside the square root:

[tex]b^{2}-4ac[/tex]

Every quadratic formula has the structure:

[tex]ax^{2} +bx+c=0[/tex]

So first, in order to meet this structure we need to add 200 to both sides so the equation is equal to 0. This gives us:

[tex]x^{2} +45x+200=0[/tex]

Our a=1, b=45 and c=200

Now we can substitute these values into the discriminant:

[tex](45)^{2} -4(1)(200)[/tex]

Solve:

[tex]2025-800=1225[/tex]

The discriminant is a positive number which means this equation will have 2 real solution. Now we just need to plug in our values into the quadratic formula to solve this equation. Quadratic formula:

[tex]x=\frac{-+/-\sqrt{b^{2}-4ac} }{2a} \\x=\frac{-45+/-\sqrt{1225} }{2}[/tex]

(Same discriminant value)

[tex]x=\frac{-45+/-35}{2}[/tex]

Now to find the two solutions, we use both signs in the equation. Solution 1:

[tex]x=\frac{-45+35}{2}[/tex]

[tex]x=\frac{-10}{2}=-5[/tex]

Our first solution is -5, now for the second:

[tex]x=\frac{-45-35}{2}\\\\ x=\frac{-80}{2}=-40[/tex]

The two solution to this equation are -5 and -40.

Hope this helped!

I NEED HELP ITS URGENT!!! I WILL MARK BRAINLIST!! LOOK AT THE IMAGE ABOVE AND DO BOTH PROBLEMS BY SHOWING THE ENTIRE WORK!!

Do NOT answer if you DONT know. This could lead to serious consequences. Ex: “idk” could lead to the monitor marking you to loose points.

PLEASE HELP

Answers

Figure 1:
x+120=180.
x=180-120
x=60


Figure 2:
a+73=180(linear pair)
a=180-73
a=103


a+x=180(alternate interior angle)
103+x=180
x=180-103
x=77

Answer:

okay well all i can explain is the image on the far left, the answer would be x=112 and then for the one on the far right (im not sure ) i think its 17

explanation: so a straight line had 180 degress and the outside angle is 126 meaning that if you do 180-126 you can find the amount of the inside angle on the bottom right. When you do this you get 54 degress. Then remember that a triangle has a total of 180 degrees all added up so the final angle inside the triagle is 80 minus the sum of the other two angles. 58+54=112 and 180-112=68. remember that 68 is the angle on the INSIDE and x is clearly on the outside meaning that you would need to do 180-68. your final answer would be 112. (and im really sorry i couldn't answer the other ones they're too blurry  :(

help plz ive been stuck on this for a while now

Answers

Answer:1.20/5

Step-by-step explanation:

Answer:

9/10 or in decimal form 0.9

Step-by-step explanation:

SOME 1 HELP WITH 11 PLZZZZ

Answers

Answer: Tiffany got 45 questions right! Hope this helped.

Step-by-step explanation:

12. A worker can pack 72 boxes of oranges in 2 hours. At this rate, how many boxes of oranges can she pack
in 8 hours?

Answers

72/2
36
36*8
Answer
288
228 because 72x4 or 36x8

Plz help me I’ll give brainliest!!

Answers

Answer:

k = 1

Step-by-step explanation:

The equation of a parabola in vertex form is

f(x) = a(x - h)² + k

where (h, k) are the coordinates of the vertex and a is a multiplier

Here (h, k ) = (4, 1 )

with k = 1

The cost of a speeding ticket is a function of each mile per hour that exceeds the speed limit multiplied by a fixed rate of $2 plus a $40 processing fee.Write the function that represents this relationship.Then determine the cost of a speedingticket for a car that has gone 50 mph in a 35 mph zone

Answers

The cost of the speeding ticket for a car that has gone 50 mph in a 35 mph zone is $70.

Describe Function?

In mathematics, a function is a relationship between two sets of numbers, known as the domain and the range, that assigns each element of the domain a unique element in the range. A function can be thought of as a machine that takes input from the domain and produces output in the range.

More formally, a function f from a set X to a set Y is denoted as f: X → Y

For example, the function f(x) = 2x assigns to each real number x a unique real number 2x. When x = 1, f(1) = 2; when x = -3, f(-3) = -6.

Functions can be represented in a variety of ways, including algebraic expressions, tables of values, and graphs. The graph of a function is a visual representation of the relationship between the input and output values, and can be used to analyze the behavior of the function.

Functions are important in many areas of mathematics, science, and engineering, and are used to model relationships between quantities, solve problems, and make predictions. They are also used in computer programming, where they are used to encapsulate behavior and modularize code.

The function that represents the relationship between the speed of a car and the cost of a speeding ticket can be expressed as:

C(x) = 2(x - L) + 40

where:

C(x) is the cost of the speeding ticket for a car traveling at speed x (in mph).

L is the speed limit (in mph).

(x - L) represents the number of miles per hour over the speed limit that the car was traveling.

The fixed rate is $2 per mile per hour over the limit, and there is a $40 processing fee added to every ticket.

To determine the cost of a speeding ticket for a car that has gone 50 mph in a 35 mph zone, we substitute the values for x and L into the formula above:

C(50) = 2(50 - 35) + 40

C(50) = 2(15) + 40

C(50) = 30 + 40

C(50) = $70

Therefore, the cost of the speeding ticket for a car that has gone 50 mph in a 35 mph zone is $70.

To know more about number visit:

https://brainly.com/question/29766862

#SPJ1

Make Sense and Persevere A submarine
descends to of its maximum depth. Then it
descends another of its maximum depth. If it
is now at 650 feet below sea level, what is its
maximum depth?

Answers

Answer: 325 feet

Explanation: you cut the amount of feet below sea level in half since it only depended twice. And you get 325

HELP ASAP I'M GIVING BRAINLIEST

Answers

Answer:

the picture is blurry is it only like that for me?

Please help me I got a bad grade and I need to redo them
Find the slope

Answers

Answer:

The first one has a slope of 4

The second one has a slope of 0.3 repeating

Step-by-step explanation:

You pick two points on the graph.

You find the x and y coordinates of each point.

You find the difference between the two x coordinates.

You find the difference between the two y coordinates.

Divide the difference in the y coordinates by the x coordinates.

If im wrong sorry owo im only in elementary school... <3

Answer:

4/6

Step-by-step explanation:

PLSS!!!! HELP!!! ME I WILL MARK U!!!!

Answers

Answer:

562

Step-by-step explanation:

Plug in the values of x, y, and z provided.

(5(5)-1)^2-2(7)

Solve.

(25-1)^2-14

(24)^2-14

576-14

562

Answer: 562

Step-by-step explanation:

(5x+y)^2-2z

first put in the known values. (5(5)+(-1))^2-2(7)

(25+-1)^2-14

Second, do 25+-1=24

(24)^2-14

Third, do 24 squared

576-14

Subtract

562

PLS HELP WITH THIS QUESTION

Answers

C would be the answer to this problem!

Answer:

I think C

Step-by-step explanation:

Look at the angle shown below.
V.
Estimate the measure of the angle.

Answers

how is anyone supposed to answer this question when you don’t shown the angle

Melanie had 3 pounds of dried fruit. She packed the dried fruit into bags of 3/8 pound each. How many bags of dry fruit does she pack?

Answers

Answer: 8 bags

Step-by-step explanation:

From the question, we are informed that Melanie had 3 pounds of dried fruit and that she packed the dried fruit into bags of 3/8 pound each.

For us to get the number of bags of dry fruit does she will pack, we have to divide 3 pounds by 3/8 pounds. This will be:

= 3 ÷ (3/8)

= 3 × (8/3)

= 8

Therefore, she will pack 8 bags of dried fruit.

Please answer this.. please

Answers

the second one and the last one

operaciones complejas raiz de -100​

Answers

Creo que 10i , si no es eso pues nose good luck hehe

Answer:

Step-by-step explanation:

10 i

es esto

y = -5x– 1
y = -5x + 7

Answers

Answer:

see below

Step-by-step explanation:

y = -5x– 1

y = -5x + 7

Since they are both equal to y, we can set them equal to each other

-5x-1 = -5x+7

Add 5x to each side

-5x-1+5x= -5x+7+5x

-1 = 7

This is not true so there is no solution

The lines are parallel with different y intercepts

The answer is “There is on solution “:)

50 points plus Brainnlest for right answer

What is the equation?
What should you replace for each given variable?
What should you solve for?

Answers

Equations: 60x + 3000 = 3,000, 600 + 150y = 3000

X: 20

Y: 10

You should solve for x & y

Q1: 0, it's already filled up to the max of 3000 pounds.

Q2: 16, 60 times 10 is 600. 3000-600 = 2400. 2400/150 = 16

I hope this helped, ask if you need more help :)

evaluate the expression when a=32 and b=7 a/4 +b

Answers

I believe your answer would be 15:)

if triangle ocd ≅ triangle JAL, then which of the following statements are true?

Answers

Answer:

I III and IV

Step-by-step explanation:

According to the SAS (Side Angle Side) postulate, the side OC is congruent to side JA, the angle COD is congruent to angle AJL, and the angle DCO is congruent to angle JAL.

Given :

Triangle OCD ≅ Triangle JAL

The given triangles can be similar through the ASA postulate. According to the Angle Side Angle postulate, two angles and included side of one triangle is congruent to the other two angles and included side of another triangle then both the triangles are congruent.

So according to the SAS (Side Angle Side) postulate, the side OC is congruent to side JA, the angle COD is congruent to angle AJL, and the angle DCO is congruent to angle JAL.

Therefore, the correct option is D).

For more information, refer to the link given below:

https://brainly.com/question/10629211

Jordan invested $97,000 in an account paying an interest rate of 1.6% compounded
monthly. Assuming no deposits or withdrawals are made, how long would it take, to
the nearest year, for the value of the account to reach $109,700?

Answers

ill give u the answer hold up i gotta do the work

Answer:

t≈ 8

Step-by-step explanation:

\text{Compounded Monthly:}

Compounded Monthly:

A=P\left(1+\frac{r}{n}\right)^{nt}

A=P(1+  

n

r

​  

)  

nt

 

Compound interest formula

A=109700\hspace{35px}P=97000\hspace{35px}r=0.016\hspace{35px}n=12

A=109700P=97000r=0.016n=12

Given values

109700=

109700=

\,\,97000\left(1+\frac{0.016}{12}\right)^{12t}

97000(1+  

12

0.016

​  

)  

12t

 

Plug in values

109700=

109700=

\,\,97000(1.0013333)^{12t}

97000(1.0013333)  

12t

 

Simplify

\frac{109700}{97000}=

97000

109700

​  

=

\,\,\frac{97000(1.0013333)^{12t}}{97000}

97000

97000(1.0013333)  

12t

 

​  

 

Divide by 97000

1.1309278=

1.1309278=

\,\,1.0013333^{12t}

1.0013333  

12t

 

\log\left(1.1309278\right)=

log(1.1309278)=

\,\,\log\left(1.0013333^{\color{blue}{12t}}\right)

log(1.0013333  

12t

)

Take the log of both sides

\log\left(1.1309278\right)=

log(1.1309278)=

\,\,\color{blue}{12t}\log\left(1.0013333\right)

12tlog(1.0013333)

Bring exponent to the front

\frac{\log\left(1.1309278\right)}{\log\left(1.0013333\right)}=

log(1.0013333)

log(1.1309278)

​  

=

\,\,\frac{12t\log\left(1.0013333\right)}{\log\left(1.0013333\right)}

log(1.0013333)

12tlog(1.0013333)

​  

 

Divide both sides by log(1.0013333)

92.3402971=

92.3402971=

\,\,12t

12t

Use calculator

\frac{92.3402971}{12}=

12

92.3402971

​  

=

\,\,\frac{12t}{12}

12

12t

​  

 

Divide by 12

7.6950248=

7.6950248=

\,\,t

t

Denise runs a 10 kilometer race (6.2 miles). Her average time per mile is shown.

Answers

Answer:

Step-by-step explanation:

8.5*6.2=52.7

The required time taken by Denise to complete the race is 52.7 minutes.

Given that,
Denise runs a 10-kilometer race (6.2 miles). Her average time per mile is 8.5 minutes per mile.

What is speed?

Speed is defined as when an object is subjected to move, the distance covered by the object in a certain time interval is the speed of that object.

What is simplification?

The process in mathematics to operate and interpret the function to make the function or expression simple or more understandable is called simplifying and the process is called simplification.

Here,
Denise runs a 10-kilometer race (6.2 miles).
Denise run 1 mile in 8.5 minutes
Time is taken by denis to run 6.2 miles =  6.2 * 8.5
                                                                  = 52.7 minutes

Thus, the required time taken by Denise to complete the race is 52.7 minutes.

Learn more about simplification here:
https://brainly.com/question/12501526

#SPJ2

help please i'll love you forever thanksssss . *btw i am going to post a lot of questions if you answer & explain I'll give you brainliest each time .*

Isabella noticed that when she doesn't study for a test, on average she makes a 45. Isabella also realizes that for every hour she studies, on average she scores an additional 7 points. Which of the following represents g, get average test score she studied for h hours?

a. 7-5h=g
b. 7+5h=g
c. 45+7h=g
d. 45-7h=g

Answers

Answer:

C. 45 + 7h = g

Step-by-step explanation:

This one would be correct because you have to add 7 points to the 45 points she would get on average without studying, without forgetting to multiply 7 by the number of hours that Isabella studied.

Example:

Let's say that Isabella studied for 3 hours. We could replace the h for the number of hours that she studied.

45 + 7(3) = g

Now you just have to solve the equation.

45 + 21 = g

Following the rules of PEMDAS, you would get rid of the parenthesis first by multiplying 7 times 3. Now you just have to add and you will have your answer for g.

g = 66

Leslie gained 5.54 pounds between her third and fourth birthdays. If she weighed 42.48 pounds when on her fourth birthday, how much did she weigh in pounds on her third birthday? Does this mean to add or subtract? Pick the correct operation and solve ONLY that one.

Answers

Answer:

We Subtract

Her weight on her third birthday = 36.94 pounds

Step-by-step explanation:

Does this mean to add or subtract? This means we add. The operation we are using is SUBTRACTION

Leslie GAINED 5.54 pounds between her third and fourth birthdays.

This means she added weight

She weighed 42.48 pounds on her Fourth birthday

Hence, Her weight in her third birthday is:

Third birthday + 5.54 pounds = 42.48

$42.48

Third birthday = 42.48 - 5.54

= 36.94 pounds

Her weight on her third birthday = 36.94 pounds.

Which line is steeper Line A or Line D? EXPLAIN. PLEASE HELP I WILL MARK BRAILIEST!!

Answers

It would be line A, because of the fact the it is rising at a faster rate.

Answer:

line A is steeper.

Line A´s slope is 1, while line d´s slope is 2/5

a
(
n
)
=
c

5
+
6
(
n

1
)
a
(
n
)
=
c
-
5
+
6
(
n
-
1
)

Answers

Answer:

what should we have to do here

I don't understand

Other Questions
Write a haikupoemexplainingwhy graphingis useful.If you areable, shareyour poemwith others. Frederick is training for a triathlon and wants to focus on swimming this week. He can swim 25 yards in 23 seconds. He set up a proportion to determine how long it would take him to swim 175 yards at this rate. Which proportions can be used to determine how long it would take him to swim 175 yards? brainliest!which word is an antonym of raucous as used in this sentence?The raucous guest was escorted from the wedding and told not to return.(rowdy isn't the answer)a. upsetb. subduedc. thirstyd. rowdy Can anyone do this question? does the surface in contact with ground affect the going motion of the car ?why? A carpenter has less than 120 minutes to spend painting furniture each day. Today, he has spent 30.5 minutes painting a desk. Now he will paint x chairs, each of which takes 12.5 minutes. What is the maximum number of chairs the carpenter can paint? they only said simplify and i guessed the answer as undefined b. If this branch of the military changes the height requirements so that all women are eligible except the shortest 1% and the tallest 2%, what are the new height requirements? What are the domain and range of f (x)= -27 PLEASE HELP!!!There are 800 students who attend Park Middle School. There are 1.5 times as many boys as there are girls. Assign variables to the unknowns. Two pounds of raw materials are needed to produce one product. The desired ending balance of raw materials is 10% of the of the next month's production needs. (Refer to your answer in Activity Sheet 05). The beginning inventory of raw materials is 4,000 pounds and raw material cost per unit is P 0.50/per pound. Which is denser a liquid or solid why? Estimate the total mass of ocean on earth If national income is $5,000 billion, compensation of employees is $1,105 billion, proprietors income is $1,520 billion, corporate profits are $490 billion, and net interest is $128 billion, then rental income is equal to What is the largest structure of the heart?atriumventricleaortavalve Which of the following explains the factors responsible for setting prices in afree-market system?A. The balance of costs and benefits.B. Supply relative to demand.C. Revenue minus costs.D. The amount of scarcity.Please help!!! The book contains 200 thin sheets of paper.The student wants to find the average mean, thickness of a sheet of paper in the bookDescribe how he can determine such a small distance using only a ruler. A driver of a car traveling at -15m/s applies the brakes, causing a uniform acceleration of +2.0m/s2. If the brakes are applied for 2.5s, what is the velocity of the car at the end of the braking period? How far has the car moved during the braking period? What algebraic expression could represent the average of 2x, x + 3, and 6x? You throw a ball straight up into the air at a velocity of 15 m/s. You want to know how high above your hand the ball will be at exactly 2.5 sec after you released it. You must SHOW ALL WORK to receive credit!!! Of the 5 motion variables - Vi, Vf, a, X, and T, identify which one you are solving for and the magnitudes and directions of all of the others that are known: Which of the motion equations is best to use to determine the height of the ball? How high is the ball above your hand at 2.5 sec after you throw it?